- PowerScore Staff
- Posts: 5972
- Joined: Mar 25, 2011
- Fri Nov 03, 2017 2:39 pm
#41164
Complete Question Explanation
(The complete setup for this game can be found here: lsat/viewtopic.php?t=8477)
The correct answer choice is (E)
The question stem adds a new condition: V Z. This new rule affects the initial diagram, producing the following controlling chain:
Answer choice (A) is incorrect because without the possibility of Z getting out of the car first, R must be the first clown to get out of the car.
Answer choice (B) is incorrect because T must exit the car no later than third.
Answer choice (C) is incorrect because the earliest Q could exit the car is fifth.
Answer choice (D) is incorrect because the latest V could get out of the car is fourth.
Answer choice (E) is the correct answer.
(The complete setup for this game can be found here: lsat/viewtopic.php?t=8477)
The correct answer choice is (E)
The question stem adds a new condition: V Z. This new rule affects the initial diagram, producing the following controlling chain:
Answer choice (A) is incorrect because without the possibility of Z getting out of the car first, R must be the first clown to get out of the car.
Answer choice (B) is incorrect because T must exit the car no later than third.
Answer choice (C) is incorrect because the earliest Q could exit the car is fifth.
Answer choice (D) is incorrect because the latest V could get out of the car is fourth.
Answer choice (E) is the correct answer.
You do not have the required permissions to view the files attached to this post.
Dave Killoran
PowerScore Test Preparation
Follow me on X/Twitter at http://twitter.com/DaveKilloran
My LSAT Articles: http://blog.powerscore.com/lsat/author/dave-killoran
PowerScore Podcast: http://www.powerscore.com/lsat/podcast/
PowerScore Test Preparation
Follow me on X/Twitter at http://twitter.com/DaveKilloran
My LSAT Articles: http://blog.powerscore.com/lsat/author/dave-killoran
PowerScore Podcast: http://www.powerscore.com/lsat/podcast/